Proof of power rule of limit laws

Click For Summary
The discussion centers on proving the Power Rule for limits, specifically that if r and s are integers with no common factor and s is not zero, then the limit of f(x) raised to the power r/s as x approaches c equals L raised to the power r/s, given L is a real number. A connection to the Sandwich Theorem is suggested, indicating that proving the limit of f squared leads to the conclusion that the limit of f raised to any integer power can be established through induction. The conversation also raises a question about the assumption that the limit of f raised to the power of 1/m exists as x approaches c. The participants seek clarity on the foundational assumptions necessary for the proof. The discussion highlights the importance of understanding limit laws in calculus.
burkley
Messages
2
Reaction score
0

Homework Statement


Power Rule: If r and s are integers with no common factor and s=/=0, then
lim(f(x))r/s = Lr/s
x\rightarrowc
provided that Lr/s is a real number. (If s is even, we assume that L>0)
How can I prove it?

Homework Equations





The Attempt at a Solution


I heard that the proof is related to The Sandwich Theorem
 
Physics news on Phys.org
From \lim_{x\rightarrow c} f(x)g(x)= \left(\lim_{x \rightarrow c}f(x)\right)\left(\lim_{x \rightarrow c}g(x) you should be able to prove that \lim_{x\rightarrow c}f^2(x)= \left(\lim_{x\rightarrow c} f(x)\right)^2. Then use induction to prove that \lim_{x\rightarrow c}f^n(x)= \lim_{x\rightarrow c}\left(f(x)\right)^n. That's the easy part.

For \lim_{x\rightarrow c} f^{1/m}(x), assuming that limit exists, define g(x)= f^{1/n}(x) and look at lim_{x\rightarrow c}g^n(x).
 
Thank you for answering me. But how can we assume that lim f^1/m(x) exists?
x->c
 
Question: A clock's minute hand has length 4 and its hour hand has length 3. What is the distance between the tips at the moment when it is increasing most rapidly?(Putnam Exam Question) Answer: Making assumption that both the hands moves at constant angular velocities, the answer is ## \sqrt{7} .## But don't you think this assumption is somewhat doubtful and wrong?

Similar threads

  • · Replies 2 ·
Replies
2
Views
2K
Replies
3
Views
2K
Replies
6
Views
2K
  • · Replies 5 ·
Replies
5
Views
2K
  • · Replies 5 ·
Replies
5
Views
2K
  • · Replies 3 ·
Replies
3
Views
957
  • · Replies 1 ·
Replies
1
Views
2K
  • · Replies 2 ·
Replies
2
Views
2K
  • · Replies 5 ·
Replies
5
Views
3K
Replies
32
Views
3K